LSAT and Law School Admissions Forum

Get expert LSAT preparation and law school admissions advice from PowerScore Test Preparation.

 afinelli
  • Posts: 26
  • Joined: Sep 05, 2011
|
#1819
I chose B for this question. I immediately shot down C because it referenced areas surrounding the city - and I assumed it would not be relevant to the argument. I see why C works - but B still throws me off. Is B not right because it does not MOST undermine the reasoning and C undermines it more? Just curious because B is pretty attractive...
Thanks for any help!
 Nikki Siclunov
PowerScore Staff
  • PowerScore Staff
  • Posts: 1362
  • Joined: Aug 02, 2011
|
#1832
The argument is making a causal conclusion:

Cause: Taxes on cigarettes
Effect: Reduction in smoking

B is incorrect, because even if consumers are more likely to continue buying cigarettes if their price increases due to higher taxes than if it does for other reasons, that does not mean that the tax on cigarettes will not necessarily lead to a reduction in smoking. In general, the suggestion that there are other (potentially better) means of accomplishing a certain objective does not mean that the means proposed (i.e. higher taxes in this case) will not work.

C should not be discounted, as this is a Weaken (not MBT) question. If C is true, it shows that higher taxes will simply shift the practice of buying cigarettes to other areas with lower taxes. That's exactly what happens in states that impose high taxes on liquor - NYC residents go to Jersey to buy their vodka :-)
 afinelli
  • Posts: 26
  • Joined: Sep 05, 2011
|
#1839
Thanks!
 jonwg5121
  • Posts: 38
  • Joined: Jun 06, 2015
|
#18888
Why would (E) not work? Is it because the antismoking education program isn't a part of the argument?

Also, so (B) would not work because it still might lead to a reduction in smoking?

Thanks!
 Steve Stein
PowerScore Staff
  • PowerScore Staff
  • Posts: 1153
  • Joined: Apr 11, 2011
|
#18907
Hi Jon,

Again, the argument is basically this:

Premise: Cigarette sales drop in cities where stiff taxes are imposed.
Conclusion: Taxes would produce the sought after reduction in smoking.

To weaken this argument, correct answer choice (C) provides that smoking in the surrounding areas usually increases after stiff taxes are imposed on cigarettes in a city. This weakens the argument that the taxes actually reduce smoking in general, because it seems as though taxes might merely push the places of purchase outside the cities.

With regard to answer choice (E), the fact that such programs might lose their funding has no bearing on the question of whether or not a tax on cigarettes would cause smoking rates to decrease. As for answer choice (B), this choice compares makes an irrelevant comparison. It doesn't matter which types of price increases (taxation or otherwise) consumers are more responsive to, because there is evidence that increased taxes do indeed reduce cigarette purchases within the city where the tax is imposed.

I hope that's helpful! Please let me know whether this is clear--thanks!

~Steve
 Franny_i
  • Posts: 12
  • Joined: Aug 07, 2018
|
#49284
Hi all. I read the comments and I too chose B for this. Maybe I'm reading too much into this because though I understand C says sales increase in surrounding areas, the conclusion states sales drop in cities. Was my mistake equating the reduction in smoguessking to the sales drop in cities? Also, C mentions sales increasing around the area but we're left to guess where or if the buyers smoke. I'm probably reading too much into it.
 Who Ray
PowerScore Staff
  • PowerScore Staff
  • Posts: 27
  • Joined: Jul 31, 2018
|
#49480
Hey Franny,

I think you might be thinking a little too hard about this question. You should not have assumed the drop in sales is the same as a drop in smoking, especially when AC C brings up other places that people could be buying cigarettes. If this was an anti fast food measure and the city shut down McDonalds but Burger King had a big jump in business, we could hardly say the measure was a success.
While the stimulus does not say whether people smoked the tobacco they bought, it also does not say that these are anti tobacco smoking activists that are pushing for these measures, only that they are anti tobacco. Therefore, the activists would be upset regardless. Furthermore, it doesn't matter where people smoke only if they smoke, because that is the intent of the antismoking campaign.

Cheers,
Who Ray
 jwheeler
  • Posts: 39
  • Joined: Aug 19, 2018
|
#59920
I had trouble determining between C and E here. My thought with E was initially that if it loses funding, that means the taxes went away. And if the taxes went away, then smoking would (likely) go back up, making the taxes ineffective towards reaching their goal.

I eventually settled on C because then my other consideration was that it doesn't necessarily say that the loss of funding was because of elimination of the tax. It could be the case that the tax remains in place, while the revenue from it goes towards something else. So my thought was that it was reading into it too much/making too many assumptions for E to be correct. Was my reasoning correct?
 Brook Miscoski
PowerScore Staff
  • PowerScore Staff
  • Posts: 418
  • Joined: Sep 13, 2018
|
#61703
jwheeler,

The goal is to weaken the stimulus--so we want to show that the taxes don't reduce cigarette smoking. Answer choice (C) points out that people just pick up their smokes out of city, and does the job.

(E) I would look at differently than you put it. (E) grants that the program is successful (reduces smoking), so that doesn't weaken the argument that the plan will reduce smoking. As you put it, you have to make your own assumption to think that smoking will increase again, since (E) leaves you with reduced, not equivalent, smoking.

Additionally, (E) is just making an observation that if sales reduce then so does the tax revenue from the sales. That doesn't mean that the tax has gone away, just that people don't buy the product as much. So, (E) doesn't suggest that there will be a recovery of sales due to the tax being removed.
 okjoannawow
  • Posts: 17
  • Joined: Mar 04, 2019
|
#63161
Hi,

I'm confused as to why D is incorrect. I understand why C is correct, seeing as it was one of my contenders. My assumption in trying to weaken the argument was that the Mayor was trying to stipulate that educational programs against smoking are useless, seeing as the tax would already lower smoking. So I chose D to attack that claim, demonstrating that the educational programs actually have good results.

Thank you!

Get the most out of your LSAT Prep Plus subscription.

Analyze and track your performance with our Testing and Analytics Package.